Search found 108 matches


I agree with C now.
I didn't consider sqrt(x) could be a fraction and then what I posted earlier regarding Stmt1 would not hold true.

by sethids

Sun Oct 05, 2008 2:53 am
Forum: Data Sufficiency
Topic: MGMAT
Replies: 6
Views: 1776

the first statement tells that when p is divided by 8,the remainder is 5... so p must be 8n+5 (where n=0,1,2,3,4..etc) p= 5,21,29,37.....etc Now when p is divided by 4 a remainder of 1 is obtained... therefore statement 1 is sufficient.. Statement 2 tells that p is the sum of squares of two positiv...

by sethids

Sat Oct 04, 2008 3:36 pm
Forum: Data Sufficiency
Topic: p is a positive odd integer,
Replies: 10
Views: 8656

Stmt1:
x^(1/2) > y
Square both sides:
x > y^2 implies x > y. Sufficient.

Stmt2:
x^3 > y.
suppose x = 2, y = 4
x^3 = 8 is > y but x < y.
Suppose x = 2, y = 1
x^3 = 8 is > y and x > y. Insufficient.

Hence answer should be A.

by sethids

Sat Oct 04, 2008 3:18 pm
Forum: Data Sufficiency
Topic: MGMAT
Replies: 6
Views: 1776

Could you please confirm Stmt1 for this question?

by sethids

Sat Oct 04, 2008 3:12 pm
Forum: Data Sufficiency
Topic: DS number system
Replies: 2
Views: 1237

Stmt1:
6 is halfway between x and y.
This means either 6 - x = y - 6
or 6 - y = x - 6
Either way we can deduce it to x + y = 12. Sufficient.

Stmt2:
y = 2x presents a set of infinite values for both the points. Hence insufficient.

Therefore A should be the answer.

by sethids

Sat Oct 04, 2008 1:19 pm
Forum: Data Sufficiency
Topic: ds2
Replies: 1
Views: 1575

From the question stem: Cost of producing x units = kx + t, therefore cost of producing 1000 units = 1000k + t. Revenue from sale of 1 unit = k + 60, therefore revenue from sale of 1000 units = 1000(k + 60) = 1000k + 60,000. Profit = 1000k + 60,000 - 1000k - t = 60000 - t. To reach this value we nee...

by sethids

Sat Oct 04, 2008 12:48 pm
Forum: Data Sufficiency
Topic: ds5
Replies: 3
Views: 1622

Stmt1: x< 8/9, x < 0.88888888... Insufficient. No information about y. Stmt2: y< 1/8, y < 0.125 Insufficient. No information about x. Combining both: If x = 0.8880 and y = 0.120 then x + y = 1.008 > 1 If x = 0.5, y = 0.1 Then x + y = 0.6 < 1 Insufficient. Hence E. What is the OA ?

by sethids

Sat Oct 04, 2008 8:15 am
Forum: Data Sufficiency
Topic: GPREP
Replies: 2
Views: 1420

Adult's Ticket = $30, Child's Ticket = $24. Number of Adult's tickets = x, Number of Child's tickets = y. We need to find (30x + 24y)/(x+y). Stmt1: y = 2x (30x + 24y)/(x+y) = (30x + 24*2x)/(x+2x) = 78x/3x =26. Sufficient. Stmt2: y = 4. No information about x (number of adult's tickets) or total tick...

by sethids

Sat Oct 04, 2008 7:32 am
Forum: Data Sufficiency
Topic: GPREP qn.
Replies: 2
Views: 1800

Question : Is |x| + |x-1| = 1 ? Stmt1: x > 0. Suppose x is 1/2. Then |x| = 1/2 and |x-1| = |1/2 - 1| = 1/2 1/2 + 1/2 = 1. If x = 3. Then |3| = 3 and |3 - 1| = 2. 3 + 2 <> 1. Hence this is insufficient. Stmt2: x<=1. Suppose x = 1. Then |1| + |1-1| = 1 + 0 = 1. If x = -3. Then |-3| + |-3 - 1| = 3 + 4 ...

by sethids

Sat Oct 04, 2008 5:55 am
Forum: Data Sufficiency
Topic: please tell me the OA...its urgent guys gmat on 11 oct
Replies: 1
Views: 915

D should be the answer. The authority plans to increase only 5% trains to accommodate 20% increase in ridership over a given period of time. Something that could support the authority's prediction that 5% more trains would be sufficient, should indicate that there is some other contributing factor. ...

by sethids

Mon Sep 29, 2008 1:23 pm
Forum: Critical Reasoning
Topic: SET6-MOOREVILLE
Replies: 9
Views: 4766

I concur. A appears to be the best choice among the available options.

What is the source of this question?

by sethids

Sat Sep 27, 2008 11:45 am
Forum: Sentence Correction
Topic: pl explain
Replies: 4
Views: 1728

Language variations still occur due to diverse ethnic and cultural heritage.

Option B makes this a thing of the past or at least suggests that language variations don't happen that way anymore. Hence is incorrect.

by sethids

Sat Sep 27, 2008 3:04 am
Forum: Sentence Correction
Topic: SC-increased popularity
Replies: 3
Views: 2405

I believe D should be the answer. A. Baltic sea sediments are not consistent with the industrial growth. It is the findings (the observation of heavy-metal deposits) which are consistent with the industrial growth. B. Indicates industrial activity is growing in the Baltic Sea sediments. C. "......

by sethids

Sat Sep 27, 2008 1:57 am
Forum: Sentence Correction
Topic: Baltic Sea SC
Replies: 2
Views: 1638

We are comparing the "effects of bacterial transformation" with "the effects of bacterial deterioration".

by sethids

Fri Sep 26, 2008 3:08 pm
Forum: Sentence Correction
Topic: SC-bacterial transformation
Replies: 4
Views: 3044

B should be the answer. A. Look at the sentence in parts. "...sometimes are used" - correct, "...these conditions can used" - incorrect. B. Preserves the meaning of the sentence. John Smith provides information on the conditions that lead women to a gynecologist and in the proces...

by sethids

Fri Sep 26, 2008 8:57 am
Forum: Sentence Correction
Topic: SC-gynecologist
Replies: 4
Views: 4084